Acute cardio Flashcards

1
Q
  1. A 62-year-old man presents to the emergency
    department (ED) with the chief concern of chest
    pain that woke him from sleep and radiates to
    his jaw. An electrocardiogram (ECG) reveals
    ST-segment depression in leads II, III, and aVF.
    His blood pressure is 112/62 mm Hg and heart rate
    is 60 beats/minute. Cardiac enzymes have been
    obtained, and the first troponin result was slightly
    positive. Preparations are under way to take the
    patient to the cardiac catheterization laboratory
    for evaluation. Which medication regimen is most
    appropriate for this patient at this time?
    A. Aspirin 325 mg, clopidogrel 600-mg loading
    dose (LD), and unfractionated heparin
    (UFH) infusion 80-unit/kg bolus, followed
    by 18 units/kg/hour and metoprolol 5 mg
    intravenously.
    B. Aspirin 81 mg; prasugrel 60-mg LD; UFH
    infusion 60-unit/kg bolus, followed by 12
    units/kg/hour; and intravenous enalaprilat.
    C. Aspirin 325 mg, ticagrelor 180-mg LD, and
    UFH infusion 60-unit/kg bolus, followed by
    12 units/kg/hour.
    D. Aspirin 81 mg, prasugrel 60-mg LD, nitroglycerin
    infusion at 10 mcg/minute, and
    bivalirudin 0.75-mg/kg bolus and 1.75-mg/kg/
    hour infusion.
A
  1. Answer: C
    This patient’s chest pain, ST-segment depression on
    ECG, and positive biomarkers for myocardial necrosis
    suggest NSTE-ACS. Because of his presentation
    characteristics, he is at a high enough risk to warrant
    cardiac catheterization (invasive strategy). This invasive
    strategy is used to determine whether occluded or
    partly occluded epicardial arteries exist, which ones
    can be intervened on, and whether to do PCI (percutaneous
    transluminal coronary angioplasty with or without
    stenting). Initial therapy for ACS usually consists of
    morphine, oxygen, nitroglycerin, and aspirin, but only
    aspirin has been shown to reduce mortality from these
    initial treatments. Aspirin should be given as soon
    as possible after hospital presentation and continued
    indefinitely, if tolerated. According to clinical trials,
    guidelines, and experience, an initial dose of 162–325
    mg is recommended (Answers B and D are not the
    best choices of dosing for an acute episode). Aspirin,
    together with a P2Y12 receptor antagonist, is indicated
    for an early invasive strategy in the management of UA/
    NSTEMI, improving outcomes. The 2014 NSTE-ACS
    guidelines give a class I recommendation for clopidogrel,
    ticagrelor, and prasugrel in ACS for patients
    undergoing PCI. The choice of which P2Y12 receptor
    antagonist to use in the ACS setting depends on patient
    presentation, contraindications, and whether PCI is
    involved; in this case, any of the three P2Y12 antagonists
    would be appropriate. The anticoagulation strategy
    treatment for ACS generally includes one anticoagulant
    (UFH, low-molecular-weight heparin, fondaparinux, or
    bivalirudin). When UFH is chosen as an anticoagulant
    strategy, the dose used for ACS is a 60-unit/kg bolus
    and a 12-unit/kg/hour infusion (Answer A is incorrect).
    Regarding dosing, bivalirudin (Answer D) would be an
    appropriate anticoagulation strategy; however, the initial
    aspirin dose should be higher, and a nitroglycerin
    drip would not be the best choice, given his right-sided
    MI (low blood pressure, low heart rate). Answer A is
    reasonable in patients without contraindications when
    hypertension or ongoing ischemia is a concern; however,
    initiating oral therapy within 24 hours is preferred
    in most patients as long as they have no signs of HF,
    evidence of low output state, increased risk of cardiogenic
    shock, or other contraindications to β-blockade.
    β-Blockers should initially be avoided in this patient,
    given his blood pressure and decreased baseline heart
    rate. An intravenous ACE inhibitor (Answer B) should
    not be given to patients within the first 24 hours of ACS
    because of the increased risk of hypotension. Answer C
    includes DAPT and an appropriate anticoagulant dose
    (Answer C is correct).
How well did you know this?
1
Not at all
2
3
4
5
Perfectly
2
Q
  1. An 81-year-old African American man (weight 90
    kg) presents to the ED with chest pressure (10/10
    on a pain scale). His ECG reveals ST-segment
    depression in the inferior leads. His medical history
    is significant for hypertension and chronic
    kidney disease. Pertinent laboratory results are troponin
    5.8 ng/L, serum creatinine (SCr) 3.7 mg/dL,
    and estimated creatinine clearance (eCrCl) 20 mL/
    minute. The patient has been given aspirin 325
    mg single dose; a nitroglycerin drip, initiated at 5
    mcg/minute, will be titrated to chest pain relief and
    blood pressure. The patient consents for cardiac
    catheterization after adequate hydration. Which
    anticoagulation strategy is most appropriate to initiate
    in this patient?
    A. Intravenous heparin 4000-unit intravenous
    bolus, followed by a 1000-unit/hour continuous
    infusion.
    B. Enoxaparin 90 mg subcutaneously every 12
    hours.
    C. Fondaparinux 2.5 mg subcutaneously daily.
    D. Bivalirudin 67.5-mg bolus, followed by a 157-
    mg/hour infusion.
A
  1. Answer: A
    The NSTE-ACS guidelines recommend the use of one
    anticoagulant during an acute event. Enoxaparin, UFH,
    and bivalirudin are all recommended as class I agents
    for the invasive management of NSTE-ACS. However,
    fondaparinux (Answer C) is not optimal because of the
    increased risk of catheter-related thrombosis associated
    with its use in the catheterization laboratory. The
    NSTE-ACS guidelines advise the use of an additional
    anticoagulant with class IIa activity (heparin or bivalirudin)
    if fondaparinux was an initial anticoagulant
    when the patient underwent intervention, whereas the
    PCI guidelines give fondaparinux a class III or harmful
    recommendation. Of the remaining three options, UFH
    (Answer A) is preferred because of its dosage and rapid
    clearance regardless of renal function. The UFH bolus
    should be limited to 4000 units, and the initial infusion
    should be limited to 1000 units/hour. Both enoxaparin
    (Answer B) and bivalirudin (Answer D) would be
    appropriate but would need to be dose adjusted, given
    this patient’s CrCl of less than 30 mL/minute. However,
    the doses in Answers B and D would be appropriate for
    patients with a normal CrCl.
How well did you know this?
1
Not at all
2
3
4
5
Perfectly
3
Q
  1. A 56-year-old man presents to the hospital with
    the chief concern of chest pain that was unrelieved
    at home with sublingual nitroglycerin. His ECG
    reveals ST-segment depression and T-wave inversion.
    Cardiac markers show an elevated troponin I.
    The cardiologist has requested that the patient go
    to the cardiac catheterization laboratory for further
    evaluation. The patient has a history of coronary
    artery disease (CAD) and had a myocardial infarction
    (MI) about 6 months ago. During his previous
    hospitalization, he was confirmed to have developed
    heparin-induced thrombocytopenia (HIT)
    after his platelet count (Plt) dropped to 40,000/
    mm3 and he had a positive ELISA (enzyme-linked
    immunosorbent assay) upon serologic testing after
    his previous catheterization. Given this patient’s
    diagnosis and history, which treatment regimen
    would be most appropriate during his cardiac
    catheterization?
    A. Tirofiban.
    B. Bivalirudin.
    C. Enoxaparin.
    D. Tenecteplase
A
  1. Answer: B
    An anticoagulant is required for PCI. Options include
    UFH, bivalirudin, and enoxaparin. Because this patient
    had a significantly low Plt with his most recent heparin
    exposure and was confirmed to have HIT, using any
    of the GP IIb/IIIa inhibitors (Answers A and C) would
    be unwise for ACS treatment because these agents are
    usually combined with UFH. Furthermore, GP IIb/
    IIIa inhibitors are antiplatelets, and the patient will still
    need an additional agent with anticoagulant activity.
    Thrombolytic therapy is not recommended for NSTEACS
    and would be inappropriate in this patient (Answer
    D is incorrect). Answer D is also incorrect because
    enoxaparin carries a 10% risk of cross-reactivity if HIT
    is suspected. Bivalirudin (Answer B), a direct thrombin
    inhibitor, would be the treatment of choice in patients
    with HIT undergoing PCI.
How well did you know this?
1
Not at all
2
3
4
5
Perfectly
4
Q
  1. A 62-year-old man presents to the ED after several
    hours of chest discomfort. His ECG reveals
    a 1- to 2-mm ST-segment elevation with positive
    troponins. He has also had increasing shortness of
    breath and lower-extremity swelling over the past
    2–3 weeks. His medical history is significant for
    tobacco use for 40 years, chronic obstructive pulmonary
    disease, diabetes, and hypertension. His
    blood pressure is 102/76 mm Hg and heart rate is
    111 beats/minute. He has rales in both lungs and
    2–3+ pitting edema in his extremities. His echocardiogram
    reveals an ejection fraction (EF) of 25%.
    After primary percutaneous coronary intervention
    (PCI), he is transferred to the cardiac intensive
    care unit. Which best describes the acute use of
    β-blocker therapy in this patient?
    A. Give 12.5 mg of oral carvedilol within the first
    24 hours.
    B. Give 5 mg of intravenous metoprolol at the
    bedside.
    C. Give 200 mg of oral metoprolol succinate at
    discharge.
    D. Give no β-blocker at this time.
A
  1. Answer: D
    β-Blocker therapy can cause HF decompensation, particularly
    when the β-blocker is titrated too quickly or
    initiated in patients who are not euvolemic. Although
    administering β-blockers within the first 24 hours is
    beneficial in STEMI, this patient has several risk factors
    that would be considered contraindications to initial
    β-blockade. This patient’s clinical condition suggests
    he is not euvolemic, and aggressive diuresis should be
    tried before a β-blocker is initiated for him. In addition,
    intravenous β-blocker therapy (Answer B) would
    place him at an even greater risk of cardiogenic shock.
    Answers A and C are inappropriate because the doses
    are fairly aggressive for a patient with marginal blood
    pressure at this time. Answer D is correct; however,
    before discharge, this patient should be reevaluated for
    the initiation of low-dose β-blocker therapy.
How well did you know this?
1
Not at all
2
3
4
5
Perfectly
5
Q
  1. A 60-year-old man (weight 75 kg) presents to
    the ED with crushing substernal chest pain and
    ST-segment elevations on ECG. He has a medical
    history of diabetes and a 40 pack-year history of
    smoking. He is taken immediately to the catheterization
    laboratory for primary PCI, and a drugeluting
    stent is placed in his left anterior descending
    artery. In addition to aspirin, which regimen
    would best maintain this patient’s stent patency?
    A. Clopidogrel 300-mg LD, followed by 75 mg
    daily for 12 months.
    B. Prasugrel 60-mg LD, followed by 10 mg daily
    for 12 months.
    C. Ticagrelor 180-mg LD, followed by 90 mg
    daily for 6 months.
    D. Clopidogrel 600-mg LD, followed by 75 mg
    daily for 6 months.
A
  1. Answer: B
    Dual antiplatelet therapy is recommended for at least 12
    months in patients presenting with ACS. Early discontinuation
    of DAPT is reasonable when the risk of morbidity
    exceeds the expected benefit (class IIa), as in the case of
    bleeding. Answers C and D do not represent the minimum
    time interval, given that the patient has no known
    risk of bleeding. Prasugrel (Answer B) would be preferable
    to clopidogrel (Answer A) in this scenario because
    it would be faster in onset; clopidogrel would take about
    6 hours for maximal platelet inhibition after a 300-mg
    LD. Subgroup analysis of a Triton-TIMI 38 comparing
    the effectiveness of prasugrel and clopidogrel showed the
    superiority of prasugrel, especially for patients presenting
    with a STEMI (Answer B is correct).
How well did you know this?
1
Not at all
2
3
4
5
Perfectly
6
Q
  1. A 60-year-old woman with New York Heart
    Association (NYHA) class IV heart failure (HF)
    (heart failure with reduced ejection fraction
    [HFrEF]) is admitted for increased shortness of
    breath and dyspnea at rest. Her extremities appear well perfused, but she has 3+ pitting edema in her
    lower extremities. Her vital signs include blood
    pressure 125/70 mm Hg, heart rate 92 beats/
    minute, and oxygen saturation (Sao2) 89% on 100%
    facemask. After initiating an intravenous diuretic,
    which intravenous agent is best to rapidly treat this
    patient’s pulmonary symptoms?
    A. Dobutamine.
    B. Milrinone.
    C. Nitroglycerin.
    D. Metoprolol.
A
  1. Answer: C
    This patient is well perfused and can be classified in
    Forrester hemodynamic subset II (warm and wet).
    Because the patient has pulmonary congestion (shortness
    of breath, dyspnea at rest), intravenous diuretics are
    first-line therapy. Nitroglycerin (Answer C) is best in
    this setting because vasodilatory agents can be used in
    conjunction with intravenous diuretics to improve acute
    pulmonary edema. When adjunctive therapy is needed
    in addition to loop diuretics, intravenous vasodilators
    should be considered over inotropic agents when blood
    pressure is adequate. Dobutamine (Answer A) and milrinone
    (Answer B) primarily increase CO, which is not
    a problem in warm and wet exacerbations. In addition,
    the adverse effects of these agents (increased mortality,
How well did you know this?
1
Not at all
2
3
4
5
Perfectly
7
Q
  1. A 75-year-old woman admitted for pneumonia
    has a history of several non–ST-segment elevation
    myocardial infarctions (NSTEMIs). She had an
    episode of sustained ventricular tachycardia (VT)
    during this hospitalization. Her corrected QT (QTc)
    interval was 380 milliseconds on the telemetry.
    Her left ventricular ejection fraction (LVEF) was
    found to be 25%. Her serum potassium and magnesium
    were 4.6 mEq/L and 2.2 mg/dL, respectively.
    Which intravenous agent is most appropriate for
    this patient’s ventricular arrhythmias?
    A. Procainamide.
    B. Metoprolol.
    C. Magnesium.
    D. Amiodarone.
A
  1. Answer: D
    This patient has a depressed LVEF less than 40%;
    therefore, her AAD therapy options are limited.
    Procainamide (Answer A) is indicated only in secondary
    prevention of sustained VT in patients with a normal
    LVEF greater than 40%; if given to this patient, it
    could worsen her HF. Metoprolol (Answer B) is indicated
    for treating patients with asymptomatic nonsustained
    VT and SVT associated with CAD. This patient
    had an episode of sustained VT; therefore, therapy
    beyond β-blockade is warranted. Her QTc interval is
    not prolonged at 380 milliseconds, and her serum magnesium
    concentration is within normal limits; thus, she
    does not need intravenous magnesium therapy (Answer
    C). Amiodarone (Answer D) is first line for patients
    without contraindications because of its efficacy and
    safety in patients with an LVEF less than 40%.
How well did you know this?
1
Not at all
2
3
4
5
Perfectly
8
Q
  1. A 53-year-old woman is admitted to the hospital
    after the worst headache she has ever had. Her
    medical history includes exertional asthma, poorly
    controlled hypertension, glaucoma, and hyperlipidemia.
    She is nonadherent to her medications
    and has not taken her prescribed blood pressure
    medications for 4 days. Vital signs include blood
    pressure 220/100 mm Hg and heart rate 65 beats/
    minute. She has retinal hemorrhaging on funduscopic
    examination. Which is most appropriate for
    this patient’s hypertensive emergency?
    A. Fenoldopam 0.1 mcg/kg/minute.
    B. Nicardipine 5 mg/hour.
    C. Labetalol 0.5 mg/minute.
    D. Enalaprilat 0.625 mg intravenously every 6
    hours.
A
  1. Answer: B
    This patient has target-organ damage from poorly
    controlled hypertension in the form of retinal hemorrhaging.
    Fenoldopam is contraindicated for treating
    hypertensive emergencies in the setting of glaucoma
    (Answer A is incorrect). Nicardipine is appropriate for
    this patient, given the details of this case (Answer B
    is correct). Although labetalol is effective for treating
    hypertensive emergency, this patient has a history of
    asthma and a low heart rate, making labetalol a lessthan-
    ideal option for treating her symptoms (Answer C
    is incorrect). The antihypertensive effects of enalaprilat
    depend on a patient’s renin activity, which is unknown
    in this case. Therefore, the blood pressure–reducing
    effects may be more difficult to control than when using
    a drug having a more consistent effect in individuals.
    In addition, the bolus nature of the drug is not ideal for
    tightly controlling blood pressure with no more than a
    25% reduction in MAP. Continuous infusion drugs are
    preferable for easier titration to effect in a hypertensive
    emergency (Answer D is incorrect).
How well did you know this?
1
Not at all
2
3
4
5
Perfectly
9
Q
  1. A 52-year-old woman has a witnessed cardiac
    arrest in a shopping mall and is resuscitated with
    an automatic external defibrillator device. On electrophysiologic
    study, she has inducible VT. Which
    is most appropriate for reducing the secondary
    incidence of sudden cardiac death (SCD)?
    A. Propafenone.
    B. Amiodarone.
    C. Implantable cardioverter-defibrillator (ICD).
    D. Metoprolol.
A
  1. Answer: C
    The Cardiac Arrest Study Hamburg trial compared ICD
    with AAD in survivors of cardiac arrest for secondary
    prevention of SCD. The propafenone (Answer A) study arm was discontinued early because of its significantly
    (61%) higher mortality rate compared with the ICD arm
    (Answer A is incorrect). Although this trial had a small
    sample size that prevented a statistically significant difference
    in total mortality in ICD-treated patients versus
    patients treated with either amiodarone or metoprolol,
    the incidence of sudden death was significantly lower in
    patients with an ICD (Answer C is correct; 33% vs. 13%;
    p=0.005). The Antiarrhythmics Versus Implantable
    Defibrillators trial also evaluated ICD implantation versus
    AAD therapy (primarily amiodarone) in survivors
    of SCD. Patients with ICDs had a significantly higher
    rate of survival than did those treated with drug therapy
    (89% vs. 82%; p<0.02), making Answer C preferable to
    all the other options (Answers B and D are incorrect).
How well did you know this?
1
Not at all
2
3
4
5
Perfectly
10
Q
  1. The Sudden Cardiac Death in Heart Failure trial
    evaluated the efficacy of amiodarone or an ICD
    versus placebo in preventing all-cause mortality
    in ischemic and nonischemic patients with NYHA
    class II and III HF. There was a 7.2% absolute risk
    reduction and a 23% relative risk reduction in allcause
    mortality at 60 months with an ICD versus
    placebo. Which best shows the number of patients
    needed to treat with an ICD to prevent one death
    versus placebo?
    A. 1.
    B. 4.
    C. 14.
    D. 43.
A
  1. Answer: C
    The number needed to treat can be calculated as 1/absolute
    risk reduction. Because the absolute risk reduction
    in mortality at 60 months was 7.2% with ICD versus placebo,
    1/0.072 would be used to calculate the number of
    patients needed to treat to prevent one death during this
    time. About 14 patients (Answer C) would need to be
    treated with ICD to prevent one death in 60 months versus
    placebo. Other calculations in this fashion, including
    relative risk reduction and 100% minus the absolute or
    relative risk reduction, provide no useful information
    for interpreting the trial results and yield an incorrect
    number of patients (Answers A, B, and D are incorrect).
How well did you know this?
1
Not at all
2
3
4
5
Perfectly
11
Q

Patient Cases
1. A 66-year-old woman (weight 70 kg) with a history of MI, hypertension, hyperlipidemia, and diabetes mellitus
presents with sudden-onset diaphoresis, nausea, vomiting, and dyspnea, followed by a bandlike upper
chest pain (8/10) radiating to her left arm. She had felt well until 1 month ago, when she noticed her typical
angina was occurring with less exertion. Her ECG reveals ST-segment depression in leads II, III, and aVF and
hyperdynamic T waves and positive cardiac enzymes. Blood pressure is 150/90 mm Hg, and all laboratory
results are normal; SCr is 1.2 mg/dL. Home medications are aspirin 81 mg/day, simvastatin 40 mg every night,
metoprolol 50 mg twice daily, and metformin 1 g twice daily. Which regimen is best for this patient?
A. Aspirin 325 mg, ticagrelor 180 mg one dose, and UFH 60-unit/kg bolus; then 12 units/kg/hour titrated
to 50–70 seconds with an early invasive approach.
B. Aspirin 325 mg and enoxaparin 70 mg subcutaneously twice daily with an early invasive approach.
C. An ischemia-guided strategy with tirofiban 25 mcg/kg; then 0.15 mg/kg/minute plus enoxaparin 80 mg
subcutaneously twice daily, aspirin 325 mg/day, and clopidogrel 300 mg one dose; then 75 mg once daily.
D. An ischemia-guided strategy with aspirin 325 mg and ticagrelor 180 mg one dose; plus UFH 70-unit/kg
bolus; then 15 units/kg/hour.

A
  1. Answer: A
    This patient’s atypical symptoms, ST-segment depression
    on ECG, and positive biomarkers for myocardial
    necrosis suggest NSTE-ACS. She has at least three risk
    factors for CAD, a history of CAD (prior MI), and positive
    troponins, which place her at high risk of future
    events. In such high-risk patients, an early invasive
    strategy (as in Answers A and B) is used to determine
    whether occluded or partly occluded arteries exist,
    which ones can be intervened on, and whether to make
    an intervention. An ischemia-guided approach, also
    called “medical management,” as in Answers C and D,
    would not be preferred because of this patient’s risk category
    (i.e., positive troponins). Dual antiplatelet therapy
    (aspirin plus a P2Y12 inhibitor) is indicated for an
    early invasive strategy in managing an NSTE-ACS. In
    patients undergoing PCI, clopidogrel, prasugrel, or ticagrelor
    is appropriate (Answer A is correct). After an initial
    bolus of 60 units/kg and an infusion of 12 units/kg/
    hour (Answer A is correct; Answer D is incorrect), UFH
    can be titrated to an aPTT of 50–70 seconds. Aspirin
    alone without a P2Y12 agent, as in Answer B, would
    not provide adequate antiplatelet therapy. Furthermore,
    enoxaparin would need to be dosed with a 30-mg intravenous
    bolus before initiating twice-daily subcutaneous
    dosing because this patient has positive troponins.
    Glycoprotein IIb/IIIa inhibitors, as in Answer C, can be
    useful in high-risk patients, typically those with positive
    troponins; however, their benefit has been shown
    mainly when UFH, not low-molecular-weight heparin,
    is given as the anticoagulant.
How well did you know this?
1
Not at all
2
3
4
5
Perfectly
12
Q

Patient Cases (Cont’d)
2. A 45-year-old patient underwent an elective percutaneous transluminal coronary angioplasty and drug-eluting
stent placement in the right coronary artery. Which best represents the minimum time DAPT should be continued?
A. 1 month.
B. 3 months.
C. 6 months.
D. 12 months.

A
  1. Answer: C
    Of importance, this patient case occurs in the context of
    elective stent placement, not after ACS. In the non-ACS
    setting, the duration of DAPT is determined by the type
    of stent placed (bare metal stent vs. drug-eluting stent).
    After elective drug-eluting stent placement, DAPT is
    recommended for at least 6 months because the risk of
    in-stent thrombosis is highest during this time (Answer
    C is correct). The recommendation is for at least 1
    month after bare metal stent placement because endothelialization
    of the stent usually occurs early, typically
    within 1 month after stenting (Answer A is incorrect).
    Bleeding risk may be a reason to consider earlier termination
    (after at least 1 month) of DAPT after bare
    metal stent placement. Although European guidelines consider 3 months of DAPT after drug-eluting stent
    placement appropriate, the recommended minimum
    according to the U.S. guidelines is 6 months (Answer
    B is incorrect). Twelve months of therapy is recommended
    after ACS (Answer D is incorrect).
How well did you know this?
1
Not at all
2
3
4
5
Perfectly
13
Q
  1. A 52-year-old man (weight 100 kg) with a history of hypertension and hypertriglyceridemia presents at a major
    university teaching hospital with a cardiac catheterization laboratory. He has had 3 hours of crushing 10/10
    substernal chest pain radiating to both arms that began while he was eating his lunch (seated), which is accompanied
    by nausea, diaphoresis, and shortness of breath. He has never before had chest pain of this character or
    intensity. He usually can walk several miles without difficulty and smokes 1.5 packs/day of cigarettes. Home
    medications are lisinopril 2.5 mg/day and aspirin 81 mg daily. Current vital signs include heart rate 68 beats/
    minute and blood pressure 178/94 mm Hg. His ECG reveals a 3-mm ST-segment elevation in leads V2–V4, I,
    and aVL. Serum chemistry values are within normal limits. The first set of cardiac markers shows positive
    troponins, 0.8 mcg/L (normal defined as less than 0.1 mcg/L). Which regimen is best for this patient’s STEMI?
    A. Reperfusion with primary PCI and stenting of occluded artery, together with tirofaban 25 mcg/kg; then
    0.15 mg/kg/minute, clopidogrel 300 mg one dose, and aspirin 325 mg one dose.
    B. Reperfusion with a reteplase 10-unit bolus twice, 30 minutes apart, plus a UFH 60-unit/kg bolus and a
    12-unit/kg/hour infusion, clopidogrel, and aspirin.
    C. Reperfusion with tenecteplase 25-mg intravenous push one dose, enoxaparin 30-mg intravenous bolus
    plus 100 mg subcutaneously twice daily, aspirin 325 mg one dose, ticagrelor 180 mg one dose, and bivalirudin
    0.75 mg/kg followed by 1.75 mg/kg/hour.
    D. Reperfusion with primary PCI with stenting, prasugrel 60 mg one dose, aspirin 325 mg one dose, and
    bivalirudin 0.75 mg/kg followed by 1.75 mg/kg/hour.
A
  1. Answer: D
    Because the patient presents to a hospital that can do a
    primary PCI with stent implantation, this is the preferred
    reperfusion strategy (Answers B and C are incorrect).
    Answer A is incorrect because an anticoagulant agent
    must be administered in addition to antiplatelet therapy.
    Reperfusion with fibrinolytic therapy (Answers B and
    C) would only be considered if PCI were expected to be
    delayed by more than 120 minutes. In addition, Answer
    C is incorrect because bivalirudin has not been studied
    with lytic therapy. Answer D – reperfusion with primary
    PCI, DAPT with aspirin and prasugrel, and dosing of
    bivalirudin as an anticoagulant strategy – is correct.
How well did you know this?
1
Not at all
2
3
4
5
Perfectly
14
Q
  1. A 76-year-old male smoker (weight 61 kg) has a history of hypertension, benign prostatic hypertrophy, and
    lower back pain. Three weeks ago, he began to have substernal chest pain with exertion (together with dyspnea),
    which radiated to both arms and was associated with nausea and diaphoresis. These episodes have
    increased in frequency to four or five times daily; they are relieved with rest. He has never had an ECG. Today,
    he awoke with 7/10 chest pain and went to the ED of a rural community hospital 2 hours later. He was acutely
    dyspneic and had ongoing pain. Home medications are aspirin 81 mg/day for 2 months, doxazosin 2 mg/day,
    and ibuprofen 800 mg three times daily. Vital signs include heart rate 42 beats/minute (sinus bradycardia) and
    blood pressure 104/48 mm Hg. Laboratory results include blood urea nitrogen (BUN) 45 mg/dL, SCr 2.5 mg/
    dL, and troponin 1.5 ng/L (normal value less than 0.1 ng/L). His ECG reveals a 3-mm ST-segment elevation.
    Aspirin, ticagrelor, and sublingual nitroglycerin were given in the ED. The nearest hospital with a catheterization
    laboratory facility is 2½ hours away. Which regimen is best?
    A. Give alteplase 15 units intravenously plus enoxaparin 30-mg intravenous bolus.
    B. Use an ischemia-guided treatment strategy with UFH 4000-unit intravenous bolus, followed by 800 units
    intravenously per hour.
    C. Give tenecteplase 35 mg intravenously plus UFH 4000-unit intravenous bolus followed by 800 units intravenously
    per hour.
    D. Transfer the patient to a facility for primary PCI.
A
  1. Answer: C
    Unlike the patient in case 3, this patient presents with a
    STEMI to a rural community hospital where the nearest
    hospital with catheterization laboratory facilities is
    more than 120 minutes away (i.e., lytics are indicated).
    He presents within the window for fibrinolytic therapy
    consideration (less than 6 hours after chest pain
    onset) and has no obvious contraindications. Because
    he is still having ischemic chest pain and ST-segment
    elevation, he should benefit from reperfusion therapy.
    He is experiencing bradycardia, which could indicate
    an occlusion above the area perfusing his sinoatrial or
    atrioventricular nodes. Enoxaparin is a treatment option
    for anticoagulant therapy given in conjunction with
    fibrinolytics, but the patient is at higher risk of bleeding
    from impaired enoxaparin clearance and needs a
    dosage adjustment. Furthermore, he is older than 75,
    beyond the age at which the intravenous bolus should be
    given, and the alteplase dosing is incomplete (Answer
    A is incorrect). Simply treating this patient conservatively
    with UFH alone in the setting of ongoing chest
    pain, shortness of breath, and pulmonary edema is not
    optimal (Answer B is incorrect). Diagnostic catheterization
    and possible PCI to determine whether an artery
    can be reperfused may be desirable but is complicated
    because the patient’s SCr is elevated (2.5 mg/dL), and
    he is in a rural hospital, where he cannot be assessed quickly enough (within 90–120 minutes) (Answer D
    is incorrect). Because of the shorter half-life and ease
    of administration of tenecteplase, tenecteplase may be
    preferable to alteplase. Clearance of UFH with tenecteplase
    is not as altered as with enoxaparin, and it would
    be a more appropriate therapy than enoxaparin in combination
    with a thrombolytic (Answer C is correct).
How well did you know this?
1
Not at all
2
3
4
5
Perfectly
15
Q

Patient Case
Questions 5–7 pertain to the following case.
A 72-year-old man is admitted to the hospital for HF decompensation. The patient has progressively increased
dyspnea when walking (now 10 ft [3 m], previously 30 ft [6 m]) and orthopnea (now four pillows, previously two
pillows), increased bilateral lower-extremity swelling (3+), 13 kg of weight gain in the past 3 weeks, and dietary
nonadherence. He has a history of idiopathic dilated cardiomyopathy (LVEF 25%, NYHA class III), paroxysmal
AF, and hyperlipidemia. Pertinent laboratory values are as follows: BNP 2300 pg/mL (0–50 pg/mL), K+ 4.9
mEq/L, BUN 32 mg/dL, SCr 2.0 mg/dL (baseline 1.9 mg/dL), aspartate aminotransferase (AST) 40 IU/L, alanine
aminotransferase 42 IU/L, INR 1.3, aPTT 42 seconds, blood pressure 108/62 mm Hg, heart rate 82 beats/minute,
and Sao2 95%. Home medications include carvedilol 12.5 mg twice daily, lisinopril 40 mg/day, furosemide 80 mg
twice daily, spironolactone 25 mg/day, and digoxin 0.125 mg/day.
5. Which regimen is best for treating his ADHF?
A. Carvedilol 25 mg twice daily.
B. Sodium nitroprusside 0.1 mcg/kg/min IV.
C. Furosemide 120 mg intravenously twice daily.
D. Milrinone 0.5 mcg/kg/minute.

A
  1. Answer: C
    This patient, who has ADHF, is receiving a β-blocker.
    Although long-term β-blockers can improve HF symptoms
    and reduce mortality, they can also worsen symptoms
    in the short term. It is recommended to keep
    the maintenance β-blocker therapy at the same or at a
    slightly lower dose compared with the outpatient therapy
    in patients with ADHF; increasing the β-blocker
    dose before reaching euvolemia might acutely worsen
    his clinical picture (Answer A is incorrect). In patients
    admitted with volume overload without substantial signs
    of reduced CO, it is reasonable to try intravenous loop
    diuretics initially (Answer C is correct). As gut edema
    increases, oral loop diuretics (notably furosemide)
    become less effective because of decreased absorption.
    Sodium nitroprusside could be useful if signs and
    symptoms of hypoperfusion were present. Because this
    patient presents mainly with warm and wet symptoms,
    diuretics would be first-line (Answer B is incorrect).
    Milrinone is an inotropic drug. Because of their adverse
    effects, inotropes are recommended in cold and wet
    exacerbations only after vasodilatory medications have
    failed (Answer D is incorrect).
How well did you know this?
1
Not at all
2
3
4
5
Perfectly
16
Q
  1. After being initiated on intravenous loop diuretics with only minimal urinary output, the patient is transferred
    to the coronary care unit for further management of diuretic-refractory decompensated HF. His Sao2 is now
    87% on a 4-L nasal cannula, and an arterial blood gas is being obtained. His blood pressure is 110/75 mm
    Hg and heart rate is 75 beats/minute. The patient’s SCr and K+ concentrations have begun to rise and are now
    2.7 mg/dL and 5.4 mmol/L, respectively. In addition to a one-time dose of intravenous chlorothiazide, which
    regimen is most appropriate for this patient?
    A. Nitroglycerin 20 mcg/minute.
    B. Sodium nitroprusside 0.3 mg/kg/minute.
    C. Dobutamine 5 mcg/kg/minute.
    D. Milrinone 0.5 mcg/kg/minute.
A
  1. Answer: A
    Intravenous vasodilators such as nitroglycerin (Answer
    A) and sodium nitroprusside (Answer B) are reasonable
    if intravenous diuretics fail and the patient progresses
    to acute pulmonary edema. Both agents rapidly cause
    venous vasodilation and reduce pulmonary filling
    pressures, which can relieve acute shortness of breath.
    Answer A, nitroglycerin, is optimal for this patient,
    given his declining renal function and the concern
    for increased risk of thiocyanate toxicity with sodium
    nitroprusside in this setting (Answer B is incorrect).
    Dobutamine is typically used in states of low CO
    decompensation and is counteracted by concomitant
    β-blocker therapy, making it a poor choice in patients
    receiving β-blockers (Answer C is incorrect). Although
    milrinone is a more acceptable inotropic agent in a
    patient receiving β-blockers, the dosing strategy is inappropriate as an initial dose (Answer D is incorrect).
    Finally, inotropes are generally reserved for patients
    when other therapies have failed.
How well did you know this?
1
Not at all
2
3
4
5
Perfectly
17
Q
  1. The patient initially responds with 2 L of urinary output overnight, and his weight decreases by 1 kg the next
    day. However, by day 5, his urinary output has diminished again, and his SCr has risen to 4.3 mg/dL. He was
    drowsy and confused this morning during rounds. His extremities are cool and cyanotic, blood pressure is
    89/58 mm Hg, and heart rate is 98 beats/minute. It is believed that he is no longer responding to his current
    regimen. A Swan-Ganz catheter is placed to determine further management. Hemodynamic values are cardiac
    index 1.5 L/minute/m2, SVR 2650 dynes/second/cm5, and PCWP 30 mm Hg. Which regimen is most appropriate
    for his current symptoms?
    A. Milrinone 0.2 mcg/kg/minute.
    B. Dobutamine 10 mcg/kg/minute.
    C. Sodium nitroprusside 0.1 mcg/kg/minute.
    D. Phenylephrine 20 mcg/minute.
A
  1. Answer: A
    Signs of a decreased CO state in HF (e.g., increased
    SCr, decreased mental status, cool extremities) suggest
    a cold and wet state, and adjunctive therapy is
    indicated. Positive inotropic agents such as milrinone
    will increase CO to maintain perfusion to vital organs.
    Milrinone will also vasodilate the peripheral vessels
    to unload the heart (lower SVR). Although dobutamine
    would be a potential choice in this patient, it is
    not recommended in patients receiving β-blockers, and
    the initial starting dose is too aggressive (Answer B is
    incorrect). Although this patient has low blood pressure,
    his elevated SVR suggests that he will tolerate
    the vasodilatory effects of milrinone as long as it is
    appropriately renally adjusted for worsening renal dysfunction
    (Answer A is correct). Nitroprusside would be
    relatively contraindicated in patients with an SBP less
    than 100 mm Hg and is absolutely contraindicated in
    patients with an SBP less than 90 mm Hg (Answer C is
    incorrect). Phenylephrine has no positive beta effects;
    therefore, it will not augment contractility. In addition,
    it will cause vasoconstriction through alpha stimulation,
    which will further increase SVR and probably
    worsen CO (Answer D is incorrect). Vasoconstrictors
    are reserved for patients in cardiogenic shock.
How well did you know this?
1
Not at all
2
3
4
5
Perfectly
18
Q

Patient Cases
Questions 8 and 9 pertain to the following case.
A 68-year-old man is admitted after an episode of syncope, with a presyncopal syndrome of seeing black spots
and dizziness before passing out. Telemetry monitor showed sustained VT for 45 seconds. His medical history
includes HF NYHA class III, LVEF 30%, two MIs, hypertension for 20 years, LV hypertrophy, DM, and diabetic
nephropathy. His medications include lisinopril 5 mg/day, furosemide 20 mg twice daily, metoprolol 25 mg twice
daily, digoxin 0.125 mg/day, glipizide 5 mg/day, atorvastatin 40 mg, and aspirin 81 mg/day. His blood pressure is
120/75 mm Hg, with heart rate 80 beats/minute, BUN 30 mg/dL, and SCr 2.2 mg/dL.
8. Which is the best therapy to initiate for conversion of his sustained VT?
A. Amiodarone 150 mg intravenously for 10 minutes, then 1 mg/minute for 6 hours, then 0.5 mg/minute.
B. Sotalol 80 mg twice daily titrated to QTc of about 450 milliseconds.
C. Dofetilide 500 mcg twice daily titrated to QTc of about 450 milliseconds.
D. Procainamide 20 mg/minute, with a maximum of 17 mg/kg.

A
  1. Answer: A
    Treatment options for sustained VT depend on concomitant
    disease states, particularly LVEF (40% cutoff). In
    a patient with LV dysfunction, class I agents such as
    procainamide are contraindicated (Answer D is incorrect).
    In a patient whose CrCl is less than 60 mL/minute,
    sotalol requires a considerable dose reduction to avoid
    an excess risk of torsades de pointes. Sotalol is not an
    effective cardioversion drug but is more useful for preventing
    future episodes of arrhythmias (maintaining
    sinus rhythm) once sinus rhythm is achieved (Answer
    B is incorrect). Dofetilide is indicated only for AF, not
    for ventricular arrhythmias; similarly, cardioversion
    rates with dofetilide are low (Answer C is incorrect).
    Amiodarone is first-line therapy for sustained VT in
    patients with severe renal insufficiency, HF, and SHD
    (Answer A is correct).
How well did you know this?
1
Not at all
2
3
4
5
Perfectly
19
Q
  1. The patient presents to the ED 3 months after amiodarone maintenance initiation (he refused ICD placement)
    after a syncopal episode, during which he lost consciousness for 30 seconds, according to witnesses. He also
    has rapid heart rate episodes during which he feels dizzy and lightheaded. He feels very warm all the time (he
    wears shorts, even though it is winter), cannot sleep, and has lost 3 kg in weight. He received a diagnosis of
    hyperthyroidism caused by amiodarone therapy. On telemetry, he has runs of nonsustained VT. Which best
    predicts the duration of amiodarone-associated hyperthyroidism in this patient?
    A. 12 hours.
    B. 1 month.
    C. 6 months.
    D. 18 months.
A
  1. Answer: C
    With the prolonged half-life of amiodarone and extensive
    fat tissue volume of distribution, hyperthyroid
    adverse effects would be expected to last for 3–5 halflives
    of the drug, which is 5–8 months (Answer C is
    correct; Answer A is incorrect). Although therapeutic
    concentrations may decrease substantially by then,
    1 month is too soon for the effects to subside (Answer
    B is incorrect). Although some iodine and amiodarone
    molecules will probably remain absorbed in fat stores
    for years, if not for life, therapeutic concentrations
    should not exist for longer than what is predicted by the
    half-life (Answer D is incorrect).
How well did you know this?
1
Not at all
2
3
4
5
Perfectly
20
Q
  1. A 64-year-old woman presents to the ED with the chief concern of palpitations. Her medical history includes
    hypertension controlled with a diuretic and an inferior-wall MI 6 months ago. She is pale and diaphoretic but
    can respond to commands. The patient’s laboratory values are within normal limits. Her vital signs include
    blood pressure 95/70 mm Hg and heart rate 145 beats/minute; telemetry shows sustained VT. Despite chronic
    use of β-blocker therapy, the patient has developed sustained VT that is successfully terminated with lidocaine.
    Subsequent electrophysiologic testing reveals inducible VT, and sotalol 80 mg orally twice daily is prescribed.
    Two hours after the second dose, the patient’s QTc is 520 milliseconds. Which regimen change would
    be most appropriate for this patient?
    A. Continue sotalol at 80 mg orally twice daily.
    B. Increase sotalol to 120 mg orally twice daily.
    C. Discontinue sotalol and initiate dofetilide 125 mcg orally twice daily.
    D. Discontinue sotalol and initiate amiodarone 400 mg orally three times daily.
A
  1. Answer: D
    This patient is having QT prolongation with sotalol,
    placing her at an elevated risk of developing lifethreatening
    torsades de pointes. Sotalol should be discontinued
    immediately (Answers A and B are incorrect).
    Given the QT prolongation that occurred with
    sotalol, the same will probably occur with dofetilide
    (Answer C is incorrect). Amiodarone is associated with
    minimal risk of torsades de pointes and therefore would
    be an appropriate alternative agent to prevent ventricular
    arrhythmias (Answer D is correct).
How well did you know this?
1
Not at all
2
3
4
5
Perfectly
21
Q

Patient Cases
11. A 68-year-old man with a history of stage 5 chronic kidney disease receiving hemodialysis, hypertension,
CAD post-MI, HFrEF, and gastroesophageal reflux disease presents with acute-onset shortness of breath and
chest pain. After his recent dialysis, he was nonadherent to medical therapy for 2 days and noticed he had
gained 2 kg in 24 hours. His baseline orthopnea worsened to sleeping sitting up in a chair for the 2 nights
before admission. He admits smoking cocaine within the past 24 hours and developed acute-onset chest tightness
with diaphoresis and nausea, and his pain was 7/10. He went to the ED, where his blood pressure was
250/120 mm Hg. He had crackles halfway up his lungs on examination, and chest radiography detected bilateral
fluffy infiltrates with prominent vessel cephalization. His ECG revealed sinus tachycardia, heart rate 122
beats/minute, and ST-segment depressions in leads 2, 3, and aVF. He was admitted for a hypertensive emergency.
Laboratory results are as follows: BUN 48 mg/dL, SCr 11.4 mg/dL, BNP 2350 pg/mL, troponin T 1.5
ng/L (less than 0.1 mcg/L), creatine kinase 227 units/L, and creatine kinase-MB 22 units/L. Which medication
is best for this patient’s hypertensive emergency?
A. Intravenous nitroglycerin 5 mcg/minute titrated to a 25% reduction in MAP.
B. Labetalol 2 mcg/minute titrated to a 50% reduction in MAP.
C. Sodium nitroprusside 0.25 mcg/kg/minute titrated to a 25% reduction in MAP.
D. Clonidine 0.1 mg orally every 2 hours as needed for a 50% reduction in MAP.

A
  1. Answer: A
    Hypertensive emergency should be treated immediately
    by no more than a 25% reduction in MAP over
    the first hour, followed by a further reduction to a blood
    pressure of 160/100 mm Hg over the next 2–6 hours.
    The patient’s comorbidities guide the optimal therapy.
    His dialysis and SCr of 11.4 mg/dL are contraindications
    to sodium nitroprusside (Answer C is incorrect)
    because of possible thiocyanate toxicity. Labetalol
    (and β-blockers in general) is controversial in patients
    who have taken cocaine, but its nonselective nature
    makes it an option; however, this patient appears to
    have decompensated HF, and a reduction of 50% initially
    is too rapid a decrease in blood pressure for safety
    (Answer B is incorrect). Clonidine is not appropriate
    for a hypertensive emergency because its oral form is
    difficult to titrate and can lead to precipitous drops in
    blood pressure beyond the goal 25% reduction and possibly
    stroke or worsening MI (Answer D is incorrect).
    Nitroglycerin is optimal, considering the patient’s lack
    of contraindications to this therapy and his evolving
    MI and symptoms of HF (Answer A is correct).
How well did you know this?
1
Not at all
2
3
4
5
Perfectly
22
Q
  1. A 56-year-old white woman with a long history of hypertension because of nonadherence and recently diagnosed
    HF (EF 35%) presents to the local ED with blood pressure 210/120 mm Hg and heart rate 105 beats/
    minute. She states that she felt a little lightheaded but that she now feels okay. She ran out of her blood pressure
    medications (including hydrochlorothiazide, carvedilol, and lisinopril) 3 days ago. Her current laboratory values
    are within normal limits. Which medication is best for this patient?
    A. Sodium nitroprusside 0.25 mcg/kg/minute titrated to a 25% reduction in MAP.
    B. Labetalol 80 mg intravenously; repeat until blood pressure is less than 120/80 mm Hg.
    C. Resumption of home medications; refer for follow-up within 2 days.
    D. Resumption of home medications; initiate amlodipine 10 mg daily; refer for follow-up in 1 week.
A
  1. Answer: C
    In an asymptomatic hypertensive crisis (without acute
    target-organ damage), giving intravenous medications,
    as in Answers A and B, and admitting the patient to
    the hospital are unnecessary (Answers A and B are
    incorrect). This patient is likely presenting because
    of recent nonadherence. Resuming her home medications
    (Answer C is correct) at this time would be most
    appropriate, with a close follow-up to ensure that her
    prescribed regimen is working. Adding a fourth agent
    (Answer D is incorrect) at this time is unnecessary,
    considering that her disease could be controlled on her
    current drug regimen if she were adherent. Follow-up
    should occur within the first few days, rather than waiting
    1 week.
How well did you know this?
1
Not at all
2
3
4
5
Perfectly
23
Q

Questions 1 and 2 pertain to the following case.
G.G. is a 56-year-old white man with type 2 diabetes. He
is a nonsmoker and is concerned about his risk of coronary
heart disease (CHD). His vital signs include blood pressure
(BP) 152/98 mm Hg (average home blood pressure 150/92
mm Hg), heart rate 70 beats/minute, and body mass index
(BMI) 26.5 kg/m2. His fasting laboratory test results today
include serum creatinine (SCr) 0.8 mg/dL, total cholesterol
(TC) 188 mg/dL, low-density lipoprotein cholesterol
(LDL) 130 mg/dL, high-density lipoprotein cholesterol
(HDL) 30 mg/dL, and triglycerides (TG) 90 mg/dL, and his urine albumin/creatinine ratio is 86.5 mg/g (previously 68 mg/g). The patient’s 10-year atherosclerotic cardiovascular
disease (ASCVD) risk is 21%.
1. Which is most appropriate to recommend for this patient’s BP control at today’s visit?

A. Amlodipine 5 mg/day.
B. Lisinopril 10 mg/day.
C. Hydrochlorothiazide 12.5 mg/day plus amlodipine
5 mg/day.
D. Chlorthalidone 12.5 mg/day plus lisinopril 10 mg/day.

A
  1. Answer: D
    The patient’s blood pressure is above his goal of less than
    140/90 mm Hg (according to the ADA; can consider less
    than 130/80 mm Hg because the 10-year ASCVD risk is
    greater than 15%) and less than 130/80 mm Hg (according
    to the ACC/AHA); thus, antihypertensive therapy should
    be initiated. The patient has stage 2 HTN according to the
    ACC/AHA guidelines; patients in this stage should be initiated
    on two medications of different classes, particularly
    if their blood pressure is more than 20/10 mm Hg above
    goal; Answers A and B are incorrect because they only
    recommend monotherapy. According to the 2021 ADA
    standards, treatment of HTN should include drug classes
    shown to reduce CV events in patients with diabetes (ACE
    inhibitors, ARBs, thiazide-like diuretics, or dihydropyridine
    CCBs), with the specific use of ACE inhibitors or
    ARBs if the patient has albuminuria or CAD. The ACC/
    AHA guidelines recommend ACE inhibitors and ARBs as
    first-line treatment for patients with diabetes in the presence
    of albuminuria, making Answer D correct. Calcium
    channel blockers and thiazide diuretics are also recommended
    as first-line agents. Answer D is correct because
    it includes both a thiazide diuretic, which improved CV
    outcomes in the ALLHAT trial, and an ACE inhibitor for
    renal protection. Answer C is incorrect because it does
    not include an ACE inhibitor or an ARB, one of which is
    needed because the patient has albuminuria.
How well did you know this?
1
Not at all
2
3
4
5
Perfectly
24
Q
  1. Which is best for primary prevention of CHD for this patient?

A. Aspirin 325 mg/day.
B. Atorvastatin 10 mg/day.
C. Aspirin 81 mg/day plus atorvastatin 40 mg/day.
D. Atorvastatin 40 mg/day plus clopidogrel 75 mg/
day.

A
  1. Answer: C
    Using the PCE risk calculator, this patient has an estimated
    10-year ASCVD risk of 21%. Because the patient is 40–75
    years old with diabetes and has an LDL of 70–189 mg/
    dL, at least moderate-intensity statin therapy is indicated.
    However, high-intensity statin therapy can be considered if
    several risk factors exist or in patients 50–75 years of age,
    according to the 2018 AHA/ACC Multisociety cholesterol
    guidelines. Similarly, the ADA guidelines state that highintensity
    statin therapy can be considered in patients with
    diabetes with several ASCVD risk factors; this patient has
    elevated blood pressure, dyslipidemia, and albuminuria
    (Answer B is incorrect). For patients with diabetes, the
    ADA recommends aspirin 75–162 mg for men 50 and older
    who have at least one other risk factor for CHD (Answer
    C is correct). If indicated, aspirin 81 mg can be used for
    primary prevention if the patient is at low risk of bleeding
    (Answer A is incorrect). Clopidogrel is recommended
    for primary prevention only if the patient has an allergy to aspirin (Answer D is incorrect). Once the patient’s lipids
    and blood pressure are better controlled, it may be appropriate
    to discontinue aspirin therapy because the benefits
    of therapy are closely balanced by risks, particularly in
    those with a lower risk of ASCVD.
How well did you know this?
1
Not at all
2
3
4
5
Perfectly
25
Q
  1. J.R. is a 55-year-old Hispanic man (weight 66 kg)
    with a medical history significant for HTN and a
    myocardial infarction (MI) 9 months ago, leading to
    placement of a bare metal stent (BMS). His current
    medications include aspirin 325 mg/day, clopidogrel 75 mg/day, atorvastatin 40 mg/day, and metoprolol succinate 50 mg/day. His vital signs are BP 145/88 mm Hg and heart rate 52 beats/minute. His laboratory test results show LDL 68 mg/dL (baseline 138 mg/dL) and SCr 1.0 mg/dL. Which is currently best for secondary prevention of CHD?

A. Decrease the aspirin dose to 81 mg/day and add
lisinopril to 5 mg/day.
B. Increase atorvastatin to 80 mg/day and increase
metoprolol succinate to 100 mg/day.
C. Decrease the aspirin dose to 81 mg/day and
increase metoprolol succinate to 100 mg/day.
D. Discontinue clopidogrel and increase atorvastatin
to 80 mg/day.

A
  1. Answer: A
    The patient’s heart rate is at less than 60 beats/minute.
    Therefore, the metoprolol dose should not be increased
    because of the risk of worsening bradycardia and heart
    block, making Answers B and C incorrect. Because the
    patient is taking a high-intensity statin with an adequate
    response in his LDL (reduction of 50% or more from
    baseline), his statin dose need not be increased, making
    Answers B and D incorrect. The patient’s blood pressure is
    not at goal (goal less than 130/80 mm Hg according to the
    2017 AHA/ACC guidelines). An ACE inhibitor guideline
    is a class I recommendation for secondary prevention in
    patients with HTN. The aspirin dose after PCI associated
    with the lowest bleeding risk is 81 mg, making Answer A
    correct. Clopidogrel is indicated for at least 1 year after
    PCI in the setting of ACS, making Answer D incorrect.
How well did you know this?
1
Not at all
2
3
4
5
Perfectly
26
Q
  1. D.P. is a 60-year-old patient who is being discharged
    from the hospital after admission for an ST-segment
    elevation MI (STEMI). D.P. received a percutaneous
    coronary intervention (PCI) with a drug-eluting stent
    (DES) and is at low risk of bleeding. Which is the best
    recommendation for this patient’s aspirin and P2Y12
    inhibitor therapy?
    A. Aspirin 325 mg indefinitely and ticagrelor for at
    least 1 year.
    B. Aspirin 325 mg for 1 month; then aspirin 81 mg
    indefinitely and clopidogrel for 6 months.
    C. Aspirin 81 mg indefinitely and clopidogrel for 1
    month.
    D. Aspirin 81 mg indefinitely and prasugrel for at
    least 1 year.
A
  1. Answer: D
    Patients who receive a DES should be treated with DAPT
    with aspirin and a P2Y12 inhibitor for at least 12 months per
    most recent guidelines. After 12 months, patients should
    receive aspirin monotherapy indefinitely. The recommended
    aspirin dose is 81 mg/day. Answer D is correct
    because it recommends both the appropriate aspirin dose
    and the appropriate DAPT duration. Answer A is incorrect
    because of the aspirin dose. Answer B is incorrect because
    of both the aspirin dose and the DAPT duration. Answer
    C is incorrect because of the DAPT duration. However, it
    is important to note that recent literature has demonstrated
    that a shorter (e.g. 1 to 3 month) duration of DAPT followed
    by monotherapy with P2Y12 inhibitor therapy may
    be noninferior to 12 months of DAPT with a lower bleeding
    risk.
27
Q
  1. T.S. is a patient who had an NSTEMI with everolimus
    DES placement 5 months ago. T.S. is scheduled
    to undergo radical prostatectomy for prostate cancer.
    He underwent a noninvasive stress test, with negative
    findings for myocardial ischemia. His current medications
    include aspirin 81 mg/day and clopidogrel 75
    mg/day. Which is the best recommendation regarding
    the patient’s dual antiplatelet therapy (DAPT)?
    A. Discontinue both aspirin and clopidogrel at least 7
    days before the procedure.
    B. Continue clopidogrel, but discontinue aspirin at
    least 5 days before the procedure.
    C. Continue aspirin, but discontinue clopidogrel at
    least 7 days before the procedure.
    D. Continue aspirin, but discontinue clopidogrel at
    least 5 days before the procedure.
A
  1. Answer: D
    Patients who have recently (less than 1 year) undergone
    DES placement are at higher risk of stent thrombosis after
    the cessation of P2Y12 inhibitor therapy. Answer D is correct.
    The 2016 ACC/AHA guideline focused update on
    the duration of DAPT states that elective noncardiac surgery
    after DES implantation in patients for whom P2Y12 inhibitor therapy will need to be discontinued can be considered
    at 3–6 months if the risk of delaying surgery is
    greater than the risk of stent thrombosis. If P2Y12 inhibitor
    therapy needs to be held in patients treated with DAPT,
    the guidelines recommend reinitiating therapy as soon as
    possible. Continuing aspirin is recommended, if possible.
    However, Answers A and C are incorrect because the
    proper timing of clopidogrel discontinuation should be 5
    days, not 7 days. Answers A and B are incorrect because
    aspirin should be continued in the setting of recent ACS.
28
Q
  1. G.F. is a 50-year-old African American woman with
    peripheral arterial disease (PAD). She has been unable
    to tolerate angiotensin-converting enzyme (ACE)
    inhibitors and angiotensin receptor blockers (ARBs) because of hyperkalemia. Her current medication regimen is amlodipine 10 mg/day, chlorthalidone 12.5 mg/ day, metoprolol succinate 25 mg/day, rosuvastatin 20 mg/day, and aspirin 81 mg/day. Her vital signs today include BP 146/82 mm Hg, repeated BP 148/85 mm Hg; heart rate 78 beats/minute; weight 69 kg (154 lb); and height 64 inches. Her laboratory tests show K 4.9 mEq/L, Na 133 mEq/L, and creatinine clearance (CrCl) 65 mL/minute. Which is the best therapeutic plan for this patient?

A. No medication changes are warranted.
B. Initiate hydralazine 25 mg three times daily.
C. Increase metoprolol succinate to 50 mg/day.
D. Increase chlorthalidone to 25 mg/day.

A
  1. Answer: C
    This patient’s blood pressure must be treated because it is
    above her blood pressure goal (goal is less than 130/80 mm
    Hg, according to the 2017 AHA/ACC guidelines) (Answer
    A is incorrect). Chlorthalidone cannot be increased
    because the patient’s sodium concentration is slightly low
    (Answer D is incorrect). Hydralazine would be a third- or
    fourth-line agent that is typically reserved for patients with
    resistant HTN and is not currently indicated (Answer B is
    incorrect). Therefore, increasing the metoprolol dose would
    be most appropriate at this time (Answer C is correct).
29
Q

Questions 7 and 8 pertain to the following case.
A.M. is a 32-year-old woman with type 1 diabetes and
HTN. Her current medication regimen is as follows:
ramipril 10 mg/day, chlorthalidone 25 mg/day, amlodipine
10 mg/ day, ethinyl estradiol 20 mcg/norethindrone 1 mg
(for the past 2 years), and insulin as directed. Her vital signs
today include BP 145/83 mm Hg, repeated BP 145/81 mm
Hg, heart rate 82 beats/minute, height 66 inches, weight 70
kg (155 pounds), and BMI 24.5 kg/m2. A.M. would prefer
not to take any more drugs, if possible.

  1. Which is the best clinical plan for A.M.?
    A. No change in therapy is currently warranted.
    B. Advise weight loss and recheck her blood pressure in 3 months.
    C. Change chlorthalidone to hydrochlorothiazide.
    D. Discuss changing her contraceptive method.
A
  1. Answer: D
    Oral contraceptives, specifically estrogen, can increase
    blood pressure, and risk can increase with duration of use.
    An alternative contraceptive without estrogen would be less
    likely to contribute to the patient’s HTN, making Answer
    D correct. Answers A and B are incorrect because the
    patient’s blood pressure requires better control, but weight
    loss is unlikely to help because the patient’s BMI is normal.
    Answer C is incorrect because hydrochlorothiazide
    is no more potent than chlorthalidone, and chlorthalidone
    reduced CV events in the ALLHAT trial.
30
Q
  1. A.M. and her husband have decided that they are ready to have children. Which is best regarding A.M.’s antihypertensive regimen?

A. No change in therapy is currently warranted.
B. Discontinue ramipril and replace it with labetalol
100 mg twice daily.
C. Increase chlorthalidone to 50 mg/day.
D. Discontinue all antihypertensive therapy.

A
  1. Answer: B
    Angiotensin-converting enzyme inhibitor therapy is contraindicated
    in pregnancy because of teratogenicity, and
    discontinuing ramipril is the most important next step,
    making Answer B correct and Answers A and C incorrect.
    Answer D is incorrect because this patient will require
    very good blood pressure control during her pregnancy,
    given that untreated HTN is associated with adverse events
    in the mother, fetus, and infant.
31
Q
  1. A.D. is a 45-year-old woman with chronic kidney
    disease (CKD) and worsening HTN. Her home BP
    readings have been in the range of 130–149 mm
    Hg (systolic) over 70–79 mm Hg (diastolic) for the
    past month. Her current antihypertensive regimen
    includes lisinopril 40 mg/day, felodipine 10 mg/day,
    and carvedilol 25 mg twice daily. Her vital signs today
    include BP 139/75 mm Hg, repeated BP 138/72 mmHg; heart rate 58 beats/minute; weight 56 kg (125 pounds); and height 62 inches. Her laboratory values are as follows: SCr 1.9 mg/dL (glomerular filtration rate [GFR] 30.1 mL/minute/1.73 m2), K 5.1 mEq/L, Na 145 mEq/L, fasting blood glucose (FBG) 97 mg/ dL, TSH 2.65 mIU/mL, and albumin/creatinine ratio 66 mg/dL. Which is the next best step to better control her BP?

A. Add spironolactone 12.5 mg/day.
B. Add furosemide 20 mg twice daily.
C. Change lisinopril to losartan 100 mg/day.
D. Increase carvedilol to 50 mg twice daily.

A
  1. Answer: B
    This patient’s blood pressure goal is less than 130/80 mm
    Hg according to the ACC/AHA guidelines, and additional
    treatment is needed. Loop diuretics (Answer B) work well for patients with renal insufficiency. Answer A is incorrect
    because this patient’s potassium concentration is above
    the recommended concentration to initiate spironolactone
    (greater than 5 mEq/L), and her GFR is too low for therapy
    to benefit. Answer C is incorrect because changing from
    lisinopril to losartan would probably not lower blood pressure.
    Answer D is incorrect because carvedilol for this
    patient is at the maximum recommended dose.
32
Q
  1. B.L. is a 62-year-old white man who has had type 2
    diabetes for the past 25 years. He is a moderate alcohol
    consumer (3 drinks per night, with occasional binges)
    and is obese (BMI 40.1 kg/m2). In the past year, he
    had an MI and 5-vessel coronary artery bypass graft
    (CABG). He has well-controlled HTN (125/70 mm
    Hg). He currently takes metformin 1000 mg twice
    daily, liraglutide 1.8 mg subcutaneously once daily,
    aspirin 81 mg/day, rosuvastatin 10 mg/day, metoprolol
    25 mg twice daily, lisinopril 40 mg/day, and nitroglycerin
    0.4 mg sublingually as needed. Fasting laboratory
    test results show TC 148 mg/dL, TG 220 mg/dL, HDL
    32 mg/dL, LDL 72 mg/dL, non-HDL 116 mg/dL, SCr
    1.3 mg/dL, Na 142 mEq/L, K 4.5 mEq/L, hemoglobin
    A1C (A1C) 6.8%, and alanine aminotransferase (ALT)
    75 IU/L (history of ALT readings of 56–92 IU/L during
    the past 3 years). His primary care physician asks
    you about his chronically elevated ALT concentration
    and about continuing his statin therapy. Which is the
    best response?
    A. Decrease rosuvastatin to 5 mg/day.
    B. Change rosuvastatin to ezetimibe 10 mg/day.
    C. Continue rosuvastatin 10 mg/day.
    D. Increase rosuvastatin to 20 mg/day.
A
  1. Answer: D
    According to the 2018 AHA/ACC Multisociety cholesterol
    guidelines, 2020 AACE/ACE consensus statement on
    cholesterol management, and the 2021 ADA standards of
    care, patients with clinical ASCVD should receive highintensity
    statin therapy. The only high-intensity statin
    therapy included in the options for this case is rosuvastatin
    20 mg/day mg/day, making Answer D correct. Answer
    C is incorrect because rosuvastatin 10 mg/day is considered
    moderate intensity. Answer A is incorrect because a
    mild, chronic ALT elevation will not be adversely affected
    by statin therapy, and lowering the dose will not likely
    improve this. Answer B is incorrect because ezetimibe
    monotherapy has not reduced morbidity or mortality in
    clinical trials to date. Additionally, per the 2020 AACE/
    ACE consensus statement, this patient would be considered
    extreme risk due to having ASCVD and diabetes, making
    the LDL goal <55 mg/dL.
33
Q
  1. A.H. is a 45-year-old African American woman with
    chronic obstructive pulmonary disease and atrial fibrillation
    for whom metoprolol was changed to verapamil
    today because of severe lung disease. Today, her BP
    is 126/78 mm Hg. She has been taking simvastatin 40
    mg/day for cholesterol for the past 4 years. Her fasting
    laboratory test results show TC 196 mg/dL, TG 85
    mg/dL, HDL 50 mg/dL, LDL 129 mg/dL, non-HDL
    146 mg/dL, SCr 1.3 mg/dL, Na 141 mEq/L, and K 4.0 mEq/L. Her baseline 10-year ASCVD risk score is
    20%. Which is best to address her lipid values?
    A. No change in therapy is currently warranted.
    B. Change simvastatin to rosuvastatin 20 mg/day.
    C. Change simvastatin to lovastatin 40 mg twice
    daily.
    D. Increase the simvastatin dose to 80 mg/day.
A
  1. Answer: B
    Answer A is incorrect; a therapy change is needed because
    simvastatin is metabolized primarily by the CYP3A4 isoenzyme
    system, and verapamil competes for this same
    metabolic pathway. Answer C is incorrect; the maximal
    dose of lovastatin has similar risks. Answer D is incorrect;
    although the coadministration of simvastatin and verapamil
    is not contraindicated, the simvastatin dose should
    be decreased to 10 mg/day to reduce the risk of rhabdomyolysis.
    Rosuvastatin is metabolized through an alternative
    pathway and does not interact with verapamil, making
    Answer B correct. In addition, high-intensity statin therapy
    is indicated for this patient, according to the 2018 AHA/
    ACC Multisociety guidelines, because her baseline 10-year
    ASCVD risk score is 20%.
34
Q
  1. P.T. is a 73-year-old Asian woman with a history of
    coronary artery disease (CAD; with stent placement
    5 years ago). She has adhered to her medication regimen
    since her stent placement without problems. Her
    primary care physician checked her CK concentration
    because of muscle pain that she described as moderate
    in severity. Her medication regimen has been stable
    for the past 3 years and includes atorvastatin 40 mg/
    day, aspirin 81 mg/day, carvedilol 6.25 mg twice daily,
    and omeprazole 20 mg as needed. Fasting laboratory
    test results show TC 135 mg/dL, TG 85 mg/ dL, HDL
    50 mg/dL, LDL 68 mg/dL, non-HDL 85 mg/dL, SCr
    1.2 mg/dL (CrCl [ideal body weight {IBW}] 32 mL/
    minute), Na 141 mEq/L, K 4.0 mEq/L, and CK 503
    U/L (normal limits 20–200 U/L). Which is the next
    best step for P.T.?
    A. Continue atorvastatin 40 mg/day and continue
    monitoring patient symptoms.
    B. Discontinue atorvastatin permanently because the
    patient is statin intolerant.
    C. Hold atorvastatin and reassess the patient’s symptoms
    in 2–4 weeks.
    D. Lower the atorvastatin dose to 20 mg every evening
    and continue monitoring patient symptoms.
A
  1. Answer: C
    This patient has moderate symptoms with a CK elevation
    less than 10 times the ULN; hence, statin therapy should be
    held and symptoms reassessed in 2–4 weeks. If symptoms
    have resolved, the patient can be rechallenged with the same statin at the same dose, the same statin at a lower dose, or
    a different statin. Answers A and D are incorrect because
    they continue atorvastatin. Answer B is incorrect because
    patients are not defined as statin intolerant until they have
    had symptoms on two or three different statins, preferably
    those with alternative metabolic pathways, at least one of
    which was at the lowest dose when the symptoms occurred.
    Answer C is correct because statin therapy is being held
    and the patient is being reassessed at an appropriate time.
35
Q
  1. J.M. is a 41-year-old African American man who arrives
    at the clinic for a follow-up. Three months ago, he was
    told for the first time that his cholesterol concentration
    was high and that he had to work on his diet and exercise.
    He is otherwise healthy, does not smoke, and takes
    no prescription medications. His BP today is 121/68
    mm Hg. Fasting laboratory test results today show TC
    262 mg/dL (down from 273 mg/dL), TG 145 mg/dL
    (down from 160 mg/dL), HDL 42 mg/dL (no change),
    LDL 191 mg/dL (down from 199 mg/dL), non-HDL
    220 mg/dL (down from 231 mg/dL), and SCr 1.0 mg/
    dL. Which is best for treating his cholesterol, according
    to the 2018 American Heart Association (AHA)/
    American College of Cardiology (ACC) Multisociety
    treatment guidelines for blood cholesterol?

A. Continue to work on diet and exercise; no
medications are needed.
B. Continue to work on diet and exercise, and initiate
atorvastatin 80 mg/day.
C. Continue to work on diet and exercise, and initiate
lovastatin 20 mg/day.
D. Continue to work on diet and exercise, and initiate
simvastatin 40 mg every evening.

A
  1. Answer: B
    Because the patient’s LDL remains greater than 190
    mg/ dL, the 2018 ACC/AHA Multisociety cholesterol
    guidelines recommend a high-intensity statin such as atorvastatin
    80 mg/day, making Answer B correct and Answer
    A incorrect. Answers C and D are incorrect because they
    recommend initiating moderate- and low-intensity statins,
    respectively.
36
Q
  1. J.T. is a 36-year-old man with heterozygous familial
    hypercholesterolemia (HeFH) who arrives at the primary
    care clinic for a follow-up. He currently takes
    atorvastatin 80 mg/day. He reports adherence to his
    medication and states that he follows a low-fat diet and
    exercises 40 minutes/day 4 days/week. Fasting laboratory
    test results today show TC 275 mg/dL (baseline
    320 mg/dL), TG 204 mg/dL (baseline 265 mg/dL),
    HDL 75 mg/dL (increased from 50 mg/dL), LDL 160
    mg/dL (down from 217 mg/dL), and non-HDL 200
    mg/dL (down from 270 mg/dL). Which would best
    treat his cholesterol according to the 2018 AHA/ACC
    Mutisociety cholestrol guidelines?
    A. Initiate alirocumab 75 mg subcutaneously every
    2 weeks.
    B. Initiate ezetimibe 10 mg/day.
    C. Initiate fenofibrate 54 mg/day.
    D. Initiate colesevelam 3750 mg/day.
A
  1. Answer: B
    This patient’s disease would be categorized into statin benefit
    group 2 according to the 2018 AHA/ACC Multisociety
    cholesterol guidelines because of a baseline LDL of 190 mg/
    dL or greater. These patients should take a high-intensity
    statin to achieve 50% or more LDL lowering. The patient
    is currently receiving maximum doses of both atorvastatin
    and has achieved around 26% LDL lowering. The guidelines
    state that adding ezetimibe is reasonable if the LDL
    remains 100 mg/dL or greater after patients have received
    maximally tolerated statin therapy; Answer B is correct.
    Answer A is incorrect because PCSK9 inhibitors are
    considered only after patients have received maximally tolerated
    statin therapy plus ezetimibe and their LDL remains
    at 100 mg/dL or greater. Fenofibrate is primarily used for
    TG lowering and can increase LDL values, making Answer
    C incorrect. Although bile acid sequestrants provide additional
    LDL lowering, they can also increase TG values and
    have a relative contraindication with TG concentrations
    greater than 200 mg/dL, making Answer D incorrect.
37
Q
  1. A 65-year-old woman who had a minor ischemic
    stroke (National Institutes of Health Stroke Scale
    [NIHSS] score of −2) 3 weeks ago presents for a
    follow-up. Her current medications include aspirin
    81 mg/day (taking before the stroke), clopidogrel 75
    mg/day, chlorthalidone 25 mg/day, lisinopril 40 mg/
    day, and atorvastatin 80 mg/day. The medical resident
    has requested an evaluation of the patient’s antiplatelet
    therapy. Which recommendation would best reduce
    the patient’s risk of recurrent stroke while minimizing
    the risks of bleeding?
    A. Continue clopidogrel and aspirin for a total of 90
    days and then continue clopidogrel indefinitely.
    B. Continue both aspirin and clopidogrel indefinitely.
    C. Discontinue both clopidogrel and aspirin and initiate
    ticagrelor 90 mg twice daily indefinitely.
    D. Discontinue clopidogrel and increase aspirin dose
    to 325 mg/day indefinitely.
A
  1. Answer: A
    The 2021 AHA/ASA secondary stroke prevention guidelines
    recommend that, for a minor ischemic stroke (NIHSS
    score 3 or less) or high-risk TIA (ABCD score 4 or greater),
    DAPT with aspirin and clopidogrel should be initiated early
    (ideally within 12–24 hours of symptom onset and at least
    within 7 days of onset), which can be continued for 21–90
    days, followed by SAPT indefinitely (Answer A is correct).
    Continuing DAPT indefinitely is not recommended because of a higher risk of bleeding (Answer B is incorrect).
    Ticagrelor has no evidence to support indefinite use
    for secondary stroke prevention (Answer C is incorrect).
    There is no evidence that increasing aspirin to 325 mg/day
    is more effective in secondary stroke prevention, but it does
    increase bleeding risk (Answer D is incorrect).
38
Q

Patient Case
1. A 59-year-old white male tobacco smoker with no significant medical history presents to the wellness clinic at
your hospital for BP, diabetes, and cholesterol screening. The fasting screening tests show BP 140/86 mm Hg, TC
196 mg/dL, LDL 110 mg/dL, HDL 50 mg/dL, TG 80 mg/dL, and FBG 80 mg/dL. He asks whether he should take
an aspirin daily to prevent CVD. Which is the best response to his question?

A. Yes, a daily aspirin 162 mg will help prevent ASCVD events.
B. Yes, a daily aspirin 81 mg will help prevent ASCVD events.
C. No, his risk of intracranial bleeding is higher than his risk of an ASCVD event.
D. No, his risk of gastrointestinal (GI) bleeding is higher than his risk of an ASCVD event.

A
  1. Answer: B
    The USPSTF recommends aspirin 81 mg/day for adults age
    50–59 years with a 10-year ASCVD risk of 10% or greater,
    calculated using the ACC/AHA PCE risk calculator (tools.
    acc.org/ASCVD-Risk-Estimator-Plus/#!/calculate/ estimate/).
    Using this calculator, this patient’s 10-year ASCVD
    risk is 14.7% (sex male; age 59 years; race white; TC 196
    mg/dL; HDL 50 mg/dL; SBP 140 mm Hg; treatment for
    high blood pressure no; diabetes no; smoker yes). In addition,
    the 2019 ACC/AHA guideline for primary prevention
    of CVD states that low-dose aspirin can be considered for
    adults age 40–70 years at higher ASCVD risk who are
    not at increased risk of bleeding. Aspirin therapy is recommended
    in this patient. Answer A is incorrect because
    the USPSTF recommends low-dose aspirin. Answer B is
    correct because 81 mg/day of aspirin is the recommended
    dose for primary prevention. Answers C and D are incorrect
    because the risk of bleeding does not outweigh the
    benefit of ASCVD reduction in this patient. However, if the
    patient’s risk factors (tobacco smoking, hypertension, dyslipidemia)
    become controlled, his ASCVD risk score will
    likely decrease; in that instance, aspirin therapy is unlikely
    to have as much of a benefit, and the risk of bleeding may
    outweigh the CV benefit.
39
Q

Patient Case
2. M.P., a 52-year-old Asian woman, presents to the clinic to discuss her heartburn symptoms. Her primary care
physician has been running late, and she was rushed back to her examination room a few minutes ago. Her BP is
151/84 mm Hg. She has no history of HTN, and the only drug she takes is over-the-counter (OTC) famotidine.
Which is the next best action to take?
A. Recheck her blood pressure after she has been seated quietly for 5 minutes.
B. Initiate hydrochlorothiazide 12.5 mg/day.
C. Initiate lisinopril 10 mg/day.
D. There is no need for concern with her BP because she does not have HTN

A
  1. Answer: A
    At least two blood pressure measurements should be made
    to accurately assess blood pressure. A blood pressure measurement
    is likely inaccurate if the patient has not been
    seated quietly for at least 5 minutes (Answer A is correct).
    Determinations for therapy can be made after accurate
    blood pressure readings have been obtained (Answers B
    and C are incorrect). This patient may have undiagnosed
    HTN (Answer D is incorrect).
40
Q

Patient Case
3. A 50-year-old African American woman with no significant medical history presents to your clinic for her BP
assessment. When she participated in a health fair the past week, she was told that her BP was “too high.” Today,
her BP is 155/85 mm Hg, with a repeated BP of 150/83 mm Hg. Her HR is 72 beats/minute. She takes vitamin D
supplementation daily and acetaminophen as needed. Laboratory test results show SCr 0.9 mg/dL, K 4.0 mEq/L,
and Na 141 mEq/L. Her BMI is 26.0 kg/m2. Which is the next best action for her?
A. Treat with education on diet and exercise only.
B. Treat with education on diet and exercise, and initiate benazepril 10 mg/day.
C. Treat with education on diet and exercise, and initiate HCTZ 12.5 mg/day.
D. Treat with education on diet and exercise, and initiate HCTZ 12.5 mg/day and amlodipine 5 mg/day.

A
  1. Answer: D
    This patient meets the criteria for stage 2 HTN according
    to the 2017 AHA/ACC HTN guidelines. At stage 2, both
    nonpharmacologic and pharmacologic therapy is recommended
    (Answer A is incorrect). Thiazide diuretics and
    CCBs are a recommended first-line therapy in African
    American patients according to the 2017 AHA guidelines,
    whereas ACE inhibitors are recommended as subsequent
    therapy (Answer B is incorrect). However, because
    the patient is more than 20 mm Hg above her SBP goal,
    the guidelines recommend two antihypertensive agents
    (Answer C is incorrect; Answer D is correct).
41
Q

Patient Case
4. A 60-year-old white man with type 2 diabetes is new to your clinic. Today, his BP is 155/78 mm Hg, with a
repeated measurement of 151/73 mm Hg. His heart rate is 80 beats/minute. He could not tolerate two different
ACE inhibitors because of cough. He currently takes metformin 850 mg three times daily, glipizide 10 mg twice
daily, hydrochlorothiazide 25 mg/day, and omeprazole as needed. Laboratory test results are as follows: SCr 1.5
mg/dL (CrCl [IBW] 54 mL/minute), A1C 6.8%, K 4.0 mEq/L, and microalbumin/creatinine 98.2 mg/g. His BMI
is 31.6 kg/m2. Which would best address his elevated blood pressure?
A. No further treatment is needed.
B. Initiate amlodipine 2.5 mg/day.
C. Initiate losartan 25 mg/day.
D. Initiate metoprolol succinate 25 mg/day.

A
  1. Answer: C
    Angiotensin receptor blockers are the best replacement
    medications for ACE inhibitors in this patient because diabetes
    is a compelling indication for their use, especially
    in patients with albuminuria. In addition, ARBs have less
    risk of cough than ACE inhibitors; therefore, they can be
    used in patients who have cough with an ACE inhibitor
    (Answer C is correct; Answers B and D are incorrect).
    Treatment is needed because the patient’s blood pressure
    is above goal (goal is less than 140/90 mm Hg according to
    the ADA [may consider less than 130/80 mm Hg if 10-year
    ASCVD risk score is 15% or greater or if patient has clinical
    ASCVD] and less than 130/80 mm Hg according to
    ACC/AHA) (Answer A is incorrect).
42
Q

Patient Case
5. A 58-year-old Hispanic woman with CAD and type 2 diabetes presents to the clinic with her home BP readings.
She is frustrated because her BP is still not at goal. She currently takes HCTZ 25 mg/day, lisinopril 40 mg/day,
amlodipine 10 mg/day, and metoprolol tartrate 25 mg twice daily. She tried clonidine but had to discontinue it
because of orthostasis. Today, her BP is 148/79 mm Hg, with a repeated measurement of 145/81 mm Hg. Her HR is
58 beats/minute. Laboratory test results today show SCr 1.2 mg/dL (CrCl [adjusted body weight] 51.8 mL/minute),
K 4.2 mEq/L, and Na 142 mEq/L. Her BMI is 27.5 kg/m2, and ejection fraction is 45%. Which regimen change,
if any, would be best for this patient?

A. Initiate spironolactone 12.5 mg/day.
B. Discontinue hydrochlorothiazide, and initiate chlorthalidone 25 mg/day.
C. Increase metoprolol to 50 mg twice daily.
D. No change in her current regimen is warranted.

A
  1. Answer: B
    This patient has resistant HTN, which is the inability to
    reach a goal blood pressure with full doses of an appropriate
    three-drug regimen that includes a diuretic. The 2018
    AHA scientific statement for managing resistant HTN
    recommends substituting an optimally dosed thiazide-like
    diuretic (chlorthalidone) for the prior diuretic (hydrochlorothiazide)
    before adding an MRA such as spironolactone
    (Answer A is incorrect; Answer B is correct). Therapy
    change is needed because her blood pressure remains
    above goal (goal is less than 130/80 mm Hg according to
    the ACC/AHA and ADA guidelines) (Answer D is incorrect).
    Metoprolol cannot be increased because of her heart
    rate (Answer C is incorrect).
43
Q

Patient Case
6. A 72-year-old white man is new to your clinic. He has not seen a health care provider for the past 7 years because
of financial issues. Today, his BP is 175/100 mm Hg, with a repeated measurement of 169/99 mm Hg. He takes
no medications, although he took a drug for his BP “years ago.” He has no symptoms of illness or of feeling bad.
Today, laboratory test results are as follows: SCr 1.6 mg/dL (CrCl [adjusted body weight] 56.4 mL/minute), K
4.0 mEq/L, and Na 142 mEq/L. His BMI is 30.2 kg/m2. Which intervention would best treat this patient’s HTN?
A. Initiate lisinopril 10 mg/day.
B. Initiate HCTZ 12.5 mg/day.
C. Initiate lisinopril/HCTZ 10/12.5 mg/day.
D. Send him to the emergency department for hypertensive emergency.

A
  1. Answer: C
    Initiation of a two-drug regimen is required for this patient
    to control his blood pressure in a reasonable time because
    he has stage 2 HTN and is more than 20/10 mm Hg above
    his BP goal of less than 130/80 mm Hg (Answer C is correct;
    Answers A and B are incorrect). This patient’s blood
    pressure is not high enough to qualify as a hypertensive
    emergency (or urgency) (Answer D is incorrect).
44
Q

Patient Case
7. A 62-year-old white man who smokes 1 pack/day presents to your clinic taking no medications. His medical history
includes an MI 5 years ago, at which time a stent was placed. The primary care physician will address the
patient’s other drug-related problems but would like your help in addressing his cholesterol status. The patient
would prefer to lower his cholesterol without medications, if possible. Fasting laboratory results are as follows:
TC 187 mg/dL, TG 157 mg/dL, HDL 43 mg/dL, LDL 113 mg/dL, non-HDL 144 mg/dL, SCr 1.0 mg/dL, ALT
25 IU/L, Na 140 mEq/L, and K 4.7 mEq/L. His BMI is 32.5 kg/m2 and BP is 115/65 mm Hg. Which is the best
recommendation to treat his cholesterol?
A. Diet, exercise, and weight loss only.
B. Diet, exercise, weight loss, and simvastatin 40 mg/day.
C. Diet, exercise, weight loss, and atorvastatin 80 mg/day.
D. Diet, exercise, weight loss, and pravastatin 20 mg/day.

A
  1. Answer: C
    Per the 2020 AACE/ACE consensus statement for cholesterol
    management, this patient would be classified as very
    high risk due to having ASCVD, and his LDL goal is <70
    mg/dL. Both the 2018 AHA/ACC Multisociety cholesterol
    guidelines and the 2020 AACE/ACE consensus statement
    recommend high-intensity statin therapy due to having
    CAD and his age less than 75 (Answer C is correct, Answer A is incorrect). Simvastatin 40 mg/day is a moderate-intensity
    statin (Answer B is incorrect), and pravastatin 20 mg/
    day is a low-intensity statin (Answer D is incorrect).
45
Q

Patient Case
8. A 62-year-old woman (height 64 inches, weight 81 kg) with a history of diabetes who underwent a three-vessel
CABG 5 years earlier completes her fasting laboratory testing. Medications have been stable since her CABG
and include simvastatin 80 mg/day, glipizide 5 mg twice daily, metoprolol SR 25 mg/day, and aspirin 81 mg/
day. During the past year, her renal function had steadily decreased (6 months ago, her SCr was 1.5 mg/dL).
Laboratory results are as follows: TC 143 mg/dL, TG 160 mg/dL, HDL 42 mg/dL, LDL 63 mg/dL, non-HDL 101
mg/dL, SCr 2.3 mg/dL, CrCl (IBW) 21.9 mL/minute, ALT 45 IU/L, Na 144 mEq/L, K 4.9 mEq/L, and A1C 7.5%.
Which is the best recommendation for this patient?
A. Add omega-3 fatty acids, with eicosapentaenoic acid (EPA)/docosahexaenoic acid (DHA) at least 1 g/day.
B. Add fenofibrate 160 mg/day.
C. Continue simvastatin 80 mg/day.
D. Change simvastatin to atorvastatin 20 mg/day.

A
  1. Answer: D
    Because this patient has ASCVD, a high-intensity statin
    would normally be recommended. However, she also has
    renal insufficiency. According to the simvastatin package
    label, patients with a CrCl of less than 30 mL/minute
    should not take simvastatin 80 mg/day, making Answer C
    incorrect. However, atorvastatin 20 mg/day would be a safe
    dose for her, making Answer D correct. Fenofibrate would
    not be beneficial because her TG concentrations are not
    greater than 500 mg/dL, and fenofibrate is contraindicated
    in more severe renal dysfunction, making Answer B incorrect.
    Adding omega-3 fatty acids would be less beneficial
    than changing the patient’s regimen to an appropriately
    dosed statin, making Answer A incorrect.
46
Q

Patient Case
9. A 50-year-old African American woman who does not smoke and has no significant medical history presents for
her annual well-woman examination. Her fasting laboratory test results are as follows: TC 157 mg/dL, TG 277
mg/dL, HDL 39 mg/dL, LDL 63 mg/dL, non-HDL 118 mg/dL, SCr 0.9 mg/dL, ALT 20 IU/L, Na 144 mEq/L, K
4.5 mEq/L, and FBG 99 mg/dL. Her BMI is 30.3 kg/m2 and BP is 110/60 mm Hg. Which is best to manage her
elevated TG?
A. Diet, exercise, and weight loss.
B. Fenofibrate 160 mg/day.
C. Gemfibrozil 600 mg twice daily.
D. Pravastatin 80 mg every evening.

A
  1. Answer: A
    Because this patient’s TG concentrations are less than 500
    mg/dL, lifestyle therapies should be a focus before drug
    therapy is initiated, making Answers B and C incorrect.
    She does not require statin therapy because her 10-year
    ASCVD risk is 1.3%, which is considered low risk per the
    2018 AHA/ACC Multisociety cholesterol guidelines; therefore,
    Answer D is incorrect. Answer A is correct because
    this patient’s TG concentrations would likely be lowered as
    a result of weight loss and lifestyle modification.
47
Q

Patient Cases
10. A 52-year-old man with a history of symptomatic atrial fibrillation, dyslipidemia, and hypothyroidism presents to
the clinic. For the past month, he has been in atrial fibrillation, with unsuccessful cardioversion. His cardiologist
initiated chronic amiodarone therapy today. His medications include simvastatin 40 mg/ day, levothyroxine 137
mcg/day, warfarin as directed, and metoprolol succinate 50 mg/day. Laboratory test results are as follows: TC
131 mg/dL, TG 100 mg/dL, HDL 32 mg/dL, LDL 79 mg/dL, non-HDL 99 mg/dL, SCr 1.1 mg/dL, CrCl 83 mL/
minute, ALT 52 IU/L, Na 142 mEq/L, K 4.8 mEq/L, INR 2.3, and TSH 4.32 mIU/mL. Which is the best treatment
recommendation for this patient?
A. Discontinue simvastatin because it cannot be used concomitantly with amiodarone.
B. Increase simvastatin to 80 mg every evening because the patient’s LDL will increase once amiodarone therapy
is begun.
C. Lower simvastatin to 20 mg every evening to reduce the risk of a drug-drug interaction.
D. Add niacin 500 mg twice daily to increase his HDL.

A
  1. Answer: C
    This patient’s simvastatin dose should be reduced to
    decrease the likelihood of a drug interaction with amiodarone
    (Answer C is correct). He requires statin therapy for
    lipid control (Answer A is incorrect). Increasing the simvastatin
    dose would increase the likelihood of an adverse
    effect because of a drug-drug interaction (Answer B is
    incorrect). Although niacin could improve his HDL, this
    is not currently his primary concern. In addition, studies
    have found no benefit to treating HDL concentrations, and
    AACE/ACE recommends that low HDL be treated with
    aggressive lifestyle modifications rather than medications
    (Answer D is incorrect).
48
Q

11.A 76-year-old man with a history of CAD and diabetes completes his fasting laboratory testing. For the past
2 months, he has been treated for a diabetic foot ulcer. While interviewing the patient, you discover that he
helped his wife tear down her jewelry display at a convention and had been feeling achy during the past week.
Medications include atorvastatin 80 mg/day, niacin 500 mg twice daily, fish oil supplement 1000 mg/ day, metoprolol
succinate 25 mg/day, lisinopril 20 mg/day, and aspirin 81 mg/day. Laboratory test results are as follows: TC
148 mg/dL, TG 146 mg/dL, HDL 37 mg/dL, LDL 82 mg/dL, non-HDL 111 mg/dL, CK 1064 U/L, SCr 1.7 mg/
dL, CrCl 37 mL/minute, ALT 36 IU/L, Na 144 mEq/L, and K 5.0 mEq/L. Which is best to address his laboratory
test results?
A. Discontinue atorvastatin because of his elevated CK. The patient will be unable to take statin therapy in the
future.
B. Hold atorvastatin because of his elevated CK, and monitor his CK.
C. Continue therapy because the elevated CK is likely caused by his diabetic foot ulcer and recent strenuous
activity.
D. Change his therapy to ezetimibe/simvastatin 10/80 mg/day because his LDL is not at goal.

A
  1. Answer: B
    Because the patient is symptomatic, statin therapy should
    be held to ascertain whether his symptoms resolve or his
    CK improves. Future CK readings will dictate whether
    statin therapy should be discontinued (Answer B is correct; 11. Answer: B
    Because the patient is symptomatic, statin therapy should
    be held to ascertain whether his symptoms resolve or his
    CK improves. Future CK readings will dictate whether
    statin therapy should be discontinued (Answer B is correct;
49
Q

Patient Case
12. A 66-year-old man presents to the primary care clinic with a medical history of HTN and an MI first occurrence
with coronary stent placement that was 4 mm in diameter 8 months ago. His current medications include aspirin
81 mg/day orally, prasugrel 10 mg/day orally, nitroglycerin 0.4-mg tablets sublingually as needed for chest pain,
metoprolol succinate 75 mg/day orally, ramipril 10 mg/day orally, and atorvastatin 20 mg/day orally. The patient
denies tobacco and alcohol use. He asks how long he will need to take prasugrel. Which is the best answer?
A. If you have a BMS, call your physician because you should be able to discontinue prasugrel now.
B. If you have a DES, call your physician because you should be able to discontinue prasugrel now.
C. You will need to take prasugrel indefinitely.
D. You will need to take prasugrel for at least 1 year after your MI and stent placement.

A
  1. Answer: D
    After placement of a BMS or a DES for ACS, a P2Y12
    inhibitor is indicated for at least 1 year, making Answer D
    correct and Answers A and B incorrect. Extended DAPT
    can be considered in patients with a DAPT score of 2
    or higher; however, this patient’s current score is 0 or 1,
    depending on the type of stent placed, making Answer C
    incorrect.
50
Q

CLINICAL PEARLS OF ACEI/ARB

A

Clinical Pearls:
– Very useful as monotherapy since they do not cause
“pseudo” tolerance as with other vasodilator
medications. Why?
– As monotherapy, ACE inhibitors may be less effective
in African Americans due to lower renin release
– Confer “renal protective effects” in diabetics by
decreasing glomerular filtration pressures

51
Q

ACEI/ARB IN PREGNANCY & DRUG INTERACTIONS

A

Pregnancy:
– Category D
– Increased infant mortality has been associated with use in all
trimesters and birth defects if used in the first 9 weeks of
gestation.
§ Drug Interactions:
– Hyperkalemia is increased if used with a potassium sparing
diuretic, aldosterone antagonist, salt-substitutes
– Allopurinol, digoxin, lithium, methotrexate medications have
¯ renal excretion (note: digoxin & lithium are narrow
therapeutic index drugs)
– NSAIDs will ¯ effectiveness of ACEi

52
Q

How Do NSAIDs Decrease the
Effectiveness of ACEi & ARBs?

A

ACEIs and ARBs inhibit efferent renal arteriolar vasoconstriction that lowers glomerular filtration pressure. NSAIDs, by inhibition of prostaglandins and bradykinin, produce vasoconstriction of the afferent renal arteriole and reduce the ability of the kidney to regulate (increase) glomerular blood flow

53
Q

Whats the different mechanism of spirinolactone & eplerenone?

A

– Competitively binds to the mineralocorticoid receptor in
the distal renal tubule to prevent the intracellular gene
transcription necessary to upregulate the critical
components for Na+ and water reabsorption.
– Spironolactone differs mechanistically from eplerenone
in that spironolactone also binds and inhibits the
glucocorticoid, progesterone and androgen receptors.
– This results in a net reduction in plasma volume that can
reduce the preload and impact stroke volume.

54
Q

moa of eplerenone

A

Mechanisms of action:
– Have antifibrotic effects that reduce the remodeling
process (likely contributing factor in RALES trial)
– The presence of a 9,11 epoxide group on eplerenone
reduces its affinity for the androgen receptor,
thereby allowing more testosterone to maintain the
balance of inhibition of breast tissue proliferation.
* This is the main reason why eplerenone causes less
gynecomastia than spironolactone

55
Q

side effects and clinical considerations

A

Spironolactone has many uses/indications
and least likely to have DDI
§ Both agents reduce mortality in HF
§ Eplerenone confers lower risk of
gynecomastia but higher risk of DDI via
CYP450 3A4 and dose dependent risk of
hyperkalemia
§ Avoid both agents with most salt
substitutes & renal failure/impairment

Side Effects:
– Hyperkalemia (eplerenone may be worse & dose
dependent)
– Gynecomastia and breast tenderness (worse worse with
spironolactone)
§ Clinical Considerations:
– Avoid salt-substitutes
– May be first line in hyperaldosteronism (↑Na+ & ↓K+)
– In HF risk of hyperkalemia greater as likely on ACE/ARB
– NSAIDs can reduce the efficacy
– Avoid in renal failure or anuria àrisk of hyperkalemia

56
Q

eplerenone uses & notes

A

HTN
§ HF (post MI;
NYHA FC II – IV)

§ Dose: 25 mg daily and titrate up to
50 mg once daily
§ Major CYP3A4 substrate
§ Dose dependent risk of hyperkalemia
§ SE: Gynecomastia (~1%)

57
Q

spironolactone uses & notes

A

Hypokalemia
§ HTN
§ HF (Class III & IV)
§ Primary
Aldosteronism
§ Cirrhosis (Ascites)
§ Acne Vulgaris
§ Hirsutism

Dose in HF: 25 mg daily (up to 50 mg
daily if tolerated). Doses in cirrhosis
can be as high as 100-400 mg/d (give
40 mg lasix for every 100 mg of
spironolactone)
§ Renal dose adjust & use caution
§ Max diuretic effect 2-3 days; max
antihypertensive effect 2-3 wks
§ SE: Gynecomastia (9%), breast pain
2%)
§ RALES trial for HF indicatio

58
Q

How do alpha2 agonist work to
lower blood pressure?

A

Mechanism of Action (cont.):
– Bind to a2 receptors (inhibitory; Gi linked G-coupled
protein receptor) on postsynaptic nerves in the
nucleus tractus solitarii (NTS) regions of the medulla
oblongata of the brain stem. Their activation mimics
the activation of the baroreceptor reflex as seen with
an increase in pressure, thus tricking the sympathetic
vasomotor center that it needs to decrease.
– This creates an reduction in sympathetic outflow and
an increase in parasympathetic outflow to the heart,
blood vessels, and kidneys.

Mechanism of Action (cont.): The net effect of this
reduce sympathetic outflow is a:
– ¯ CO [because of a reduction in both heart rate and
stroke volume (from a relaxation of venous capacitance
vessels)]
* Affected most in the supine position when sympathetic outflow
to the vasculature is already low.
– ¯ TPR (or afterload)
* Affected most upon standing when the sympathetic outflow to
the vasculature increases; why see orthostasis.
– ¯ Renin release from the juxtaglomerular cells of the
nephron – this is likely to be minimal due to
compensatory mechanisms that get activated from the
reductions in CO and TPR.

59
Q

Which alpha2 agonist can be used to wean
patients off of the ventilator in the ICU?

A

Dexmedetomidine (Precedex):
– Indicated for ICU sedation and procedural sedation
– Doesn’t suppress the respiratory drive as much as
other agents àaid in weaning from ventilator
* Used for up to 24 hrs
– Side Effect:
* Hypotension but at HIGH doses and rapid infusions can
stimulate peripheral alpha 2b receptors and can cause
vasoconstriction and HTN

60
Q

Which alpha2 agonist can be used to treat
clozapine induced sialorrhea?

A

§ Clonidine is known to cause dry mucous
membranes and especially dry mouth
– It used off-label to help treat the effects of clozapine
associated drooling

61
Q

Why is a history of medication
noncompliance a concern for patients
taking clonidine?

A

§ Clonidine (Catapres):
– Immediate release formulation is greatest risk due to
shorter half-life compared to other agents
– This is the rational for the TTS (transdermal
therapeutic system)
* Apply to skin every 7 days

62
Q

How do alpha2 agonists worsen depression
and erectile dysfunction?

A

Alpha2 agonistsàpenetrate BBB
– Reduces sympathetic tone on all major organs
innervated by SNS
– Known to WORSEN both depression and erectile
dysfunction
* Erectile dysfunction is controlled by both parasympathetic
and sympathetic nervous system

63
Q

Which abnormal lab test has been
associated with the use of one of the
alpha2 agonists?

A

§ Methyldopa (main one):
– + Coomb’s test (10-30%) and about 5% may develop
hemolytic anemia
* May be due to methyldopa altering the Rh antigens in the
surface of RBCs resulting in autoimmune activation
through a process called, “molecular mimicry”